9 svar
91 visningar
Amnadhanon 45
Postad: 22 apr 2023 15:39

Bestäm polspänning U

Hej! Jag försöker förstå denna uppgift men förstår inte riktigt hur facit menar. 
vad menas med att ” för seriekopplade motstånd gäller att spänningen över motstånden är proportionell mot deras resistanter” 

vad betyder det att det är proportionellt mot deras resistaner? 
Bifogar uppgiften o facit 

uppgift 9102 

Pieter Kuiper Online 7229
Postad: 22 apr 2023 15:52 Redigerad: 22 apr 2023 15:54
Amnadhanon skrev:

vad menas med att ” för seriekopplade motstånd gäller att spänningen över motstånden är proportionell mot deras resistanser” 

Det är Ohms lag (eftersom det är samma ström genom seriekopplade motstånd).

Amnadhanon 45
Postad: 22 apr 2023 16:37

Hur menar du? Kan du förklara med med hjälp av formler? Vore tacksam för det🙏🏻 gärna förklaring hur du löser uppgiften med om du kan :)

Amnadhanon 45
Postad: 22 apr 2023 16:37
Pieter Kuiper skrev:
Amnadhanon skrev:

vad menas med att ” för seriekopplade motstånd gäller att spänningen över motstånden är proportionell mot deras resistanser” 

Det är Ohms lag (eftersom det är samma ström genom seriekopplade motstånd).

Hur menar du? Kan du förklara med med hjälp av formler? Vore tacksam för det🙏🏻 gärna förklaring hur du löser uppgiften med om du kan :)

Pieter Kuiper Online 7229
Postad: 22 apr 2023 17:03 Redigerad: 22 apr 2023 17:04
Amnadhanon skrev:
Pieter Kuiper skrev:
Amnadhanon skrev:

vad menas med att ” för seriekopplade motstånd gäller att spänningen över motstånden är proportionell mot deras resistanser” 

Det är Ohms lag (eftersom det är samma ström genom seriekopplade motstånd).

Hur menar du? Kan du förklara med med hjälp av formler? 

Ohms lag som formel är U=I RU = I \ R men det vet du nog.

"Vore tacksam för det🙏🏻 gärna förklaring hur du löser uppgiften med om du kan :) "

Du har ju en lösning. Men jag skulle kanske ha resonerat med strömmar istället: eftersom strömmen genom det ensamma motståndet är dubbelt så stor som genom de två parallellkopplade är spänningsfallet också dubbelt så stort: 12 volt. Det ger förstås samma svar.

Amnadhanon 45
Postad: 22 apr 2023 18:54
Pieter Kuiper skrev:
Amnadhanon skrev:
Pieter Kuiper skrev:
Amnadhanon skrev:

vad menas med att ” för seriekopplade motstånd gäller att spänningen över motstånden är proportionell mot deras resistanser” 

Det är Ohms lag (eftersom det är samma ström genom seriekopplade motstånd).

Hur menar du? Kan du förklara med med hjälp av formler? 

Ohms lag som formel är U=I RU = I \ R men det vet du nog.

"Vore tacksam för det🙏🏻 gärna förklaring hur du löser uppgiften med om du kan :) "

Du har ju en lösning. Men jag skulle kanske ha resonerat med strömmar istället: eftersom strömmen genom det ensamma motståndet är dubbelt så stor som genom de två parallellkopplade är spänningsfallet också dubbelt så stort: 12 volt. Det ger förstås samma svar.

Amnadhanon 45
Postad: 22 apr 2023 18:57
Pieter Kuiper skrev:
Amnadhanon skrev:
Pieter Kuiper skrev:
Amnadhanon skrev:

vad menas med att ” för seriekopplade motstånd gäller att spänningen över motstånden är proportionell mot deras resistanser” 

Det är Ohms lag (eftersom det är samma ström genom seriekopplade motstånd).

Hur menar du? Kan du förklara med med hjälp av formler? 

Ohms lag som formel är U=I RU = I \ R men det vet du nog.

"Vore tacksam för det🙏🏻 gärna förklaring hur du löser uppgiften med om du kan :) "

Du har ju en lösning. Men jag skulle kanske ha resonerat med strömmar istället: eftersom strömmen genom det ensamma motståndet är dubbelt så stor som genom de två parallellkopplade är spänningsfallet också dubbelt så stort: 12 volt. Det ger förstås samma svar.

En fråga, hur kan punkt A ha positiv potential? I området innan parallellkopplingarna är det 0 eftersom det är är jordat och eftersom man går igenom ett motstånd med strömriktningen så kommer potentialen minska dvs 0- motståndet för parallellkopplingarna och då bör väl A punkten ha en negativ potential? 

Pieter Kuiper Online 7229
Postad: 22 apr 2023 20:10 Redigerad: 22 apr 2023 20:11
Amnadhanon skrev:

En fråga, hur kan punkt A ha positiv potential? 

Det har du helt rätt i.

Det är ett fel i boken. Sådant händer eftersom den som gör figurerna för tryck inte är författaren. Sedan är det lätt att missa vid korrekturläsning.

Amnadhanon 45
Postad: 22 apr 2023 20:12
Pieter Kuiper skrev:
Amnadhanon skrev:

En fråga, hur kan punkt A ha positiv potential? 

Det har du helt rätt i.

Det är ett fel i boken. Sådant händer eftersom den som gör figurerna för tryck inte är författaren. Sedan är det lätt att missa vid korrekturläsning.

Jahaaa jag tänkte väl det men hur ska man då lista ut att en motståndare har resistans 6? Jag försökte nämligen lösa det såhär :

Pieter Kuiper Online 7229
Postad: 22 apr 2023 20:57 Redigerad: 22 apr 2023 20:58
Amnadhanon skrev:
hur ska man då lista ut att en motståndare har resistans 6?  

Så här borde symbolen för batteriet ha varit ritat:

Sedan ritade jag in strömmarna.

Strömmarnas och motståndens värde behöver man inte veta för att se att U = 18 volt.

Svara Avbryt
Close